LSAT and Law School Admissions Forum

Get expert LSAT preparation and law school admissions advice from PowerScore Test Preparation.

User avatar
 Dave Killoran
PowerScore Staff
  • PowerScore Staff
  • Posts: 5852
  • Joined: Mar 25, 2011
|
#43596
Complete Question Explanation
(The complete setup for this game can be found here: lsat/viewtopic.php?t=1755)

The correct answer choice is (B)

Answer choices (D) and (E) can be eliminated because the maximum number of conservatives voting for Datalog is one.

Answer choice (C) can be eliminated because if all three liberals vote for Datalog then the two conservatives must vote against Datalog.

Answer choice (A) is incorrect because this scenario would mean that two conservatives and a liberal voted against Datalog, and it would then follow that the two moderates voted against Datalog, a violation of the question stem.

Thus, answer choice (B) is correct.

Get the most out of your LSAT Prep Plus subscription.

Analyze and track your performance with our Testing and Analytics Package.